download scanned papers here - Paper 1 Paper II


UPSC PRELIMS 2017 PAPER I -- ANSWER KEYS


BIRD'S EYEVIEW
  1. 2017, a big challenge : The Prelims for 2017 proved to be a big challenge to anyone – fresh or experienced – who expected a set pattern based on the immediate previous year (2016), rather than being prepared for anything and everything – remember that “uncertainty is the key theme in major competitive exams”. The 2017 papers (both I and II) were not only tough, they were twisted, some questions were ambiguous and required many esoteric facts to crack. This will directly affect the cutoffs. Lesson : Long term tapasya, going in-depth, no keep hitting more, discretion, practice.
  2. Summary of major changes :
    1. Topicwise balance changed – International CA dropped by half, S & T dropped by half, Polity grew more than 3 times, Indian CA grew to triple.
    2. Toughness level went up hugely
    3. Almost no direct questions at all!
    4. Micro level stuff asked
    5. Closely placed options
    6. Trying to “hit more” backfired, totally
    7. Only long term prep helped – no mugging up of “lists”
  3. No direct questions – almost none! Since 2016 (and to some extent 2015) were current affairs heavy, and many questions could be solved ONLY by elimination of options, and since many questions were very direct requiring little more than having remembered (or even read) the concept earlier, that expectation was built into the serious aspirants – a big mistake. The first few minutes shocked those who had geared up for a “certain strategy that is bound to click”. Had they approached it with an open mind, maybe they could have stayed calm. Lesson : It is only a relative performance test, not absolute.
  4. Topicwise balance changed : The paper was dominated by questions from Polity (22 questions) and Government Schemes (12 questions – last year 13). Some topics did retain the 2016 flavour: Environment and ecology, Modern history, Indian Economy, Geography (9 compared to 6). It is worth nothing that merely cramming facts and CA was of limited use, as the questions focussed on testing the student's grasp over the underlying topics. Lesson : Don’t expect anything.
  5. Easiness vapourised : In 2016, direct questions on OBOR, TPP, Indian Navy, Neem-coated urea, EODB, Rivers, IMF, UDAY etc. made life easy (and cutoffs higher). This year, anxious students were left scratching their heads trying to find such “sitters”. There were few.
  6. Government servants should know micro of Schemes : It seems UPSC was clear that if you want to serve the government one day, you better know the ins and outs of all schemes thoroughly. The questions were tough, with micro level stuff asked.
  7. Questions on World Affairs, International Institutions and Treaties, were specific, and none could be answered on the basis of just general reading. India and World Geography was an area of importance with 9 questions. Some of the questions on these areas were not very specific but focussed on understanding of concepts.
  8. India's Independence struggle was given the usual importance this year (7 / 7). Though the questions appeared to be simple, some of them had close options which made the job tricky, while others were straightforward.
  9. Our Support : At PT's IAS Academy, we have tried to incorporate everything possible on as many topics as possible. We wish our students great luck in the results!
  10. Ambiguous answers : There were at least 4 to 5 questions with ambiguous answers ( Butterfly / Vote / IVC / gypsum etc.). This should not happen in a test like UPSC Civil Services entrance. Maybe everyone will benefit through grace marks.
  11. At this stage, we would like to suggest students to avoid getting into any speculation regarding the "Cut-Offs" for Prelims. A broad guideline could be General Category – 95 to 102 Marks, OBC – almost same and SC/ST – 85 to 92 marks. If you are anywhere close to these, get cracking for Mains preparations. Do not sulk, do not lose hope. Strange things can happen. All the best!


TOPICWISE DISTRIBUTION



DETAILED QUESTIONWISE SOLUTIONS (Set B)


1. The object of the Butler Committee of 1927 was to

  1. Define the jurisdiction of the Central and Provincial Governments.
  2. Define the powers of the Secretary of State of India.
  3. Impose censorship on national press.
  4. Improve the relationship between the Government of India and the Indian States

Sol. Ans (d) A question from "Modern Indian History" and "India’s independence movement". The Indian states committee appointed a committee under the Chairmanship of Sir Harcourt Butler (popularly called ‘the Butler Committee’) to investigate and clarify the relationship between the paramount power and the Princes of Princely States in AD 1927. The committee visited 16 States and submitted its report in 1929. It recommended (a) The relationship of the paramount power with the state was not merely a contractual relationship, but a living, growing relationship shaped by the circumstances and policy, resting on the mixture of history and theory, (b) British paramountacy preserve the princely state, (c) State should not be transferred without their own agreement to a relationship with a new government in British India responsible to an Indian legislature. Hence answer is (d).


2. The term ‘Domestic Content Requirement’ is sometimes seen in the news with reference to

  1. Developing solar power production in our country
  2. Granting licenses to foreign T.V. channels in our country
  3. Exporting our food products to other countries
  4. Permitting foreign educational institutions to set up their campuses in our country

Sol. Ans (a) A question from "Current Affairs". It is clear that "domestic content requirement" must be a rule regarding products made by foreigners in India (or by Indians somewhere else). Option (c) cannot be, as it is related to exporting to other countries, and option (d) is absurd. Option (b) is interesting due to “content”! But the correct answer is Option (a) where The intent is to promote the local manufacturing of the components of solar generation equipment which includes the cells and modules. Also, the World Trade Organization’s Appellate Body in 2016 had declared domestic content requirement (DCRs) in India’s Solar Mission illegal. Hence answer is (a). (Reference link: http://www.mnre.gov.in/solar-mission/jnnsm/introduction-2/ )


3. Consider the following statements:

  1. The Nuclear Security Summits are periodically held under the aegis of the United Nations.
  2. The International Panel on Fissile Materials is an organ of International Atomic Energy Agency.

Which of the statements given above is/are correct ?

  1. 1 only
  2. 2 only
  3. Both 1 and 2
  4. Neither 1 nor 2

Sol. Ans (d) A question from "Global Current Affairs". In 2009, U.S. President Obama delivered a speech in Prague in which he called nuclear terrorism one of the greatest threats to international security. As his brainchild, Obama hosted the first Nuclear Security Summit (NSS) in Washington DC in 2010 (hence Statement 1 is wrong, hence (a) and (c) are wrong, and either (b) or (d) will be our answer). The International Panel on Fissile Materials (IPFM), established 2006, is a group of independent nuclear experts and not under IAEA. So 2 is also wrong. Hence answer is (d). This was a peculiar question, requiring specific knowledge of international affairs. (Reference links: http://www.nss2016.org/ and https://en.wikipedia.org/wiki/International_Panel_on_Fissile_Materials )


4. Who among the following can join the National Pension System (NPS) ?

    1. Resident Indian citizens only
    2. Persons of age from 21 to 55 only
    3. All State Government employees joining the services after the date of notification by the respective State Governments
    4. All Central Government employees including those of Armed Forces joining the services on or after 1 st April 2004

Sol. Ans (c) A question from "Government Schemes". All details available on official site, here - https://india.gov.in/spotlight/national-pension-system-retirement-plan-all#nps3 Initially, NPS was introduced for the new government recruits (except armed forces). With effect from 1st May, 2009, NPS was provided for all citizens of the country including the unorganised sector workers on voluntary basis. Option (a) and (b) are ruled out, as conditions mention “All citizens of India between the age of 18 and 60 years”. This means NRIs can apply too! NPS is applicable to all new employees of Central Government service (except Armed Forces) and Central Autonomous Bodies joining Government service on or after 1st January 2004. So (d) is also ruled out. Hence answer is (c). This was a very hard question, as micro details were asked.


5. With reference to river Teesta, consider the following statements:

  1. The source of river Teesta is the same as that of Brahmaputra but it flows through Sikkim.
  2. River Rangeet originates in Sikkim and it is a tributary of river Teesta.
  3. River Teesta flows into Bay of Bengal on the border of India and Bangladesh.

Which of the statements given above is/are correct?

  1. 1 and 3 only
  2. 2 only
  3. 2 and 3 only
  4. 1, 2 and 3

Sol. Ans (b) A question from "Geography" and "Current Affairs" rolled into one! Although Teesta had to come (India-Bangladesh water sharing disputes and visit of PM Sheikh Hasina to India), this was a very hard question, as micro details were asked. The Brahmaputra River originates on the Angsi Glacier in Himalayas in Burang County of Tibet, whereas Teesta originates from the Pahunri (or Teesta Kangse) glacier. So Statement 1 is wrong. The perennial river Rangeet originates in the Himalayan mountains in West Sikkim district, and forms the boundary between Sikkim and Darjeeling district (a hot current issue), and Statement 2 is correct. Teesta is a 309 km long river and forms the border between India and Bangladesh BUT does not merge into Bay of Bengal at the border so (3) is wrong. Hence answer is (b). We have given useful resources on Rivers, and Teesta issue, here https://ias.pteducation.com/RiversofIndiaandWorld.aspx and here http://saar.bodhibooster.com/2017/04/India-Bangladesh-relations-Teesta-river-Farakka-barrage-Sheikh-Hasina-Awami-League-1971-war.html


6. Consider the following statements:

  1. In tropical regions, Zika virus disease is transmitted by the same mosquito that transmits dengue.
  2. Sexual transmission of Zika virus disease is possible.

Which of the statements given above is/are correct?

  1. 1 only
  2. 2 only
  3. Both 1 and 2
  4. Neither 1 nor 2

Sol. Ans (c) A question from "Current Affairs" and "Health/Biology" rolled into one. Zika fever is mainly spread via the bite of mosquitoes of the Aedes type, which also spreads the Dengue virus. Hence, Statement 1 is correct. Zika virus can also spread by sexual transmission from infected men to their partners. Hence Statement 2 is also correct. So answer is (c). (Reference link: https://www.cdc.gov/zika/prevention/sexual-transmission-prevention.html)


7. Consider the following statements:

  1. The Standard Mark of Bureau of Indian Standards (BIS) is mandatory for automotive tires and tubes.
  2. AGMARK is a quality Certification Mark issued by the Food and Agricultural Organisation (FAO).

Which of the statements given above is/are correct?

  1. 1 only
  2. 2 only
  3. Both 1 and 2
  4. Neither 1 nor 2

Sol. Ans (a) A question from "Current Affairs" and "Nomenclatures". Food and Agricultural Organisation (FAO) is an international organisation, but Agmark is Indian, so Statement 2 is clearly wrong. Details of the Agmark Certification Scheme : Directorate of Marketing & Inspection (DMI) in the Department of Agriculture, Cooperation and Farmers Welfare, Ministry of Agriculture, Cooperation & Farmers Welfare is implementing the provisions of Agricultural Produce (Grading & Marking) Act, 1937. Statement 1 is correct – the relevant IS 13098 Automotive vehicles –Tubes for pneumatic tyres and IS 15627 Automotive vehicles– Pneumatic tyres for two and three-wheeled motor vehicles. (Relevant links are https://en.wikipedia.org/wiki/Agmark and
http://www.bis.org.in/cert/ProdUnManCert.asp)
So answer is (a).


8. What is/are the advantage/advantages of implementing the ‘National Agriculture Market’ scheme?

  1. It is a pan-India electronic trading portal for agricultural commodities.
  2. It provides the farmers access to the nationwide market, with prices commensurate with the quality of their produce.

Select the correct answer using the code given below:

  1. 1 only
  2. 2 only
  3. Both 1 and 2
  4. Neither 1 nor 2

Sol. Ans (c) A question from "Agriculture" and "Government Schemes". The National Agriculture Market (NAM or eNAM) is an attempt by the Modi government to solve a big problem of Indian farmers – getting the right price for their produce. It is a pan-India electronic trading portal which networks the existing APMC mandis to create a unified national market for agricultural commodities. It gives an access to a nationwide market for the farmer, with prices commensurate with quality of his produce and online payment and availability of better quality produce and at more reasonable prices to the consumer. So both statements 1 and 2 are correct. However, the correct wording in the question should have been “features” and not “advantages”. (Relevant link http://www.enam.gov.in/NAM/home/about_nam.html#) So answer is (c).


9. With reference to the ‘National Intellectual Property Rights Policy’, consider the following statements:

  1. It reiterates India’s commitment to the Doha Development Agenda and the TRIPS Agreement.
  2. Department of Industrial Policy and Promotion is the nodal agency for regulating intellectual property rights in India.

Which of the above statements is/are correct?

  1. 1 only
  2. 2 only
  3. Both 1 and 2
  4. Neither 1 nor 2

Sol. Ans (c) A question from "Constitution and Law" and "Global trade". IPR is the most contests area for India, when it comes to MNCs pushing their products in, especially in pharma and other areas. The NIPR was approved by the Union Cabinet in May 2016, with the slogan “The National Intellectual Property Rights (IPR) Policy will endeavor for a “Creative India; Innovative India | Rachnatmak Bharat, Abhinav Bharat”. It is entirely compliant with the WTO’s agreement on TRIPS (Trade Related Intellectual Property Rights). It suggests making the department of industrial policy and promotion (DIPP) the nodal agency for all IPR issue. It also reiterates India’s commitment to the Doha Development Agenda. So both statements 1 and 2 are correct. (Relevant link http://pib.nic.in/newsite/PrintRelease.aspx?relid=145338) So answer is (c).


10. According to the Wildlife [Protection] Act, 1972, which of the following animals cannot be hunted by any person except under some provisions provided by law?

  1. Gharial
  2. Indian wild ass
  3. Wild Buffalo

Select the correct answer using the code given below:

  1. 1 only
  2. 2 and 2 only
  3. 1 and 3 only
  4. 1, 2 and 3

Sol. Ans (d) A question from "Environment Ecology and Climate Change". All animals listed are protect by the Wildlife Protection Act of 1972. Gharial is protected so 1 is right. Indian wild ass is protected so 2 is right, and the wild buffalo is also protect so 3 is right. (Relevant link : http://envfor.nic.in/legis/wildlife/wildlife2s1.pdf and http://vindhyabachao.org/wildlife_guidelines/schedule_species_mammals.pdf ) If you open the list, it’s a long one, and a student is expected to remember it! A nearly impossible task? But the animals mentioned are the more “popular” ones. The answer is (d).


11. Which of the following statements is/are true of the Fundamental Duties of an Indian citizen?

  1. A legislative process has been provided to enforce these duties.
  2. They are correlative to legal duties.

Select the correct answer using the code given below:

  1. 1 only
  2. 2 only
  3. Both 1 and 2
  4. Neither 1 nor 2

Sol. Ans (d) A question from "Constitution of India". Fundamental duties set out in Part IV–A of the Constitution are suggestive in nature, and one cannot move a court of law demanding they being enforced. Legal duties are compulsory actions for a citizen, but fundamental duties are not. They are expected as moral obligations of “good citizens” but that’s about it! concern individuals and the nation. Citizens are morally obligated by the Constitution to perform these duties. They are without any legal sanction in case of their violation or non-compliance. (Relevant link : http://www.supremecourtcases.com/index2.php?option=com_content&itemid=1&do_pdf=1&id=21859) The answer is (d).


12. Consider the following pairs:

  1. Radhakanta Deb- First President of the British Indian Association.
  2. Gazulu Lakshminarasu Chetty – Founder of the Madras Mahajana Sabha
  3. Surendranath Banerjee – Founder of the Indian Association

Which of the above pairs is/are correctly matched?

  1. 1 only
  2. 1 and 3 only
  3. 2 and 3 only
  4. 1 , 2 and 3

Sol. Ans (b) A question from "Modern Indian history" and "Constitution of India". We always study names in these modules, and are expected to remember as many as possible. Prior to the INC taking hold, there were many social /cultural organizations and this question explored that. Sir Surendranath Banerjee was one of the earliest Indian political leaders during the British Raj, who founded the Indian National Association. So 3 is correct. In May 1884, M. Veeraraghavachariar, G. Subramania Iyer and P. Anandacharlu established the Madras Mahajana Sabha. So 2 is wrong. Now check options : (a) cannot be right. Since 2 is wrong, (c) and (d) are wrong. Hence, the correct answer is (b). Even if you did not know about 1, the question can be cracked.
(Relevant links : https://en.wikipedia.org/wiki/Surendranath_Banerjee and https://en.wikipedia.org/wiki/Madras_Mahajana_Sabha ) The answer is (b).


13. Which one of the following objectives is not embodied in the Preamble to the Constitution of India?

  1. Liberty of thought
  2. Economic liberty
  3. Liberty of expression
  4. Liberty of belief

Sol. Ans (b) A simple and straight question from the "Constitution of India". You have to memorise the Preamble in any case, right! If you did, this was a sitter to be cracked in 10 seconds. Remember “WE, THE PEOPLE OF INDIA, having solemnly resolved to constitute India into a SOVEREIGN, SOCIALIST, SECULAR, DEMOCRATIC REPUBLIC and to secure to all its citizens: JUSTICE, social, economic and political; LIBERTY of thought, expression, belief, faith and worship; …. ” (Reference Link: http://indiacode.nic.in/coiweb/coifiles/preamble.htm ) The direct answer is (b).


14. With reference to ‘Quality Council of India (QCI)’, consider the following statements:

  1. QCI was set up jointly by the Government of Indian and the Indian Industry.
  2. Chairman of QCI is appointed by the Prime Minister on the recommendations of the industry to the Government.

Which of the above statements is/are correct?

  1. 1 only
  2. 2 only
  3. Both 1 and 2
  4. Neither 1 nor 2

Sol. Ans (c) A question from "Current Affairs" and "Governance". The Quality Council of India (QCI) was set up jointly by the Government of India and the Indian Industry, so Statement 1 is correct. The chairman of this organisation is indeed appointed by the Prime Minister on recommendations of the industry to the government. So Statement 2 is correct. This was a tough question, related to corporates and manufacturing, and given the issues related to quality, it’s important. (Reference Link: SOURCE: http://www.qcin.org/about-qci.php ) The direct answer is (c).


15. What is the purpose of setting up of Small Finance Banks (SFBs) in India?

  1. To supply credit to small business units.
  2. To supply credit to small and marginal farmers.
  3. To encourage young entrepreneurs to set up business particularly in rural areas.

Select the correct answer using the code given below:

  1. 1 and 2 only
  2. 2 and 3 only
  3. 1 and 3 only
  4. 1, 2 and 3

Sol. Ans (a) The aim for these is to provide financial inclusion to sections of the economy not being served by other banks. These sectors are : small business units, small and marginal farmers, micro and small industries and unorganised sector entities. So Statement 1 is indeed correct. But it does not involve encouraging young entrepreneurs to set up business as their main motive is financial inclusion not start-up promotion (there are other programmes for that). So Statement 3 is wrong. By option elimination we see answer is (a). If you knew that 3 is wrong, then all options get eliminated directly! (Reference Link: https://en.wikipedia.org/wiki/Small_finance_bank ) The direct answer is (a)


16. With reference to ‘Asia-Pacific Ministerial Conference on Housing and Urban Development [APMCHUD],consider the following statements:

  1. The first APMCHUD was held in India in 2006 on the theme ‘Emerging Urban Forms’ – Policy Response and Governance Structure’.
  2. India host all the Annual Ministerial Conferences in partnership with ADB, APEC and ASEAN.

Which of the statements given above is/are correct?

  1. 1 only
  2. 2 only
  3. Both 1 and 2
  4. Neither 1 nor 2

Sol. Ans (d) This was a tough one, and was based on “Current Affairs”. The stress on urban housing and development being given by the present government explains such a question. The first APMCHUD was held in 2006 in India on the theme, “A vision for sustaining urbanisation in Asia-Pacific by 2020” so Statement 1 is wrong. So either (b) or (d) is correct. India does not host all the Annual Meetings! Hence, (d) is correct. (Reference link: http://www.apmchud.com/# )


17. Democracy’s superior virtue lies in the fact that it calls into activity

  1. the intelligence and character of ordinary men and women.
  2. the methods for strengthening executive leadership
  3. a superior individual with dynamism and vision.
  4. a band of dedicated party workers.

Sol. Ans (a) This was not a direct one, but could be cracked. A question based on “Current Affairs”, “Constitution of India” and “General understanding of issues”. Option (b) is wrong as only leadership cannot be the basis of democracy which is “of, for, by the people”. Option (d) is clearly wrong as it is focusing only on Party workers (political parties). Options (a) and (c) are to be compared now – we see that (a) relates to the common citizens. Hence, (a) is correct.


18. Which of the following is a most likely consequence of implementing the 'Unified Payments Interface [UPI]'?

  1. Mobile wallets will not be necessary for online payments.
  2. Digital currency will totally replace the physical currency in about two decades
  3. FDI inflows will drastically increase.
  4. Direct transfer of subsidies to poor people will become very effective.

Sol. Ans (a) This was an easy one. A question based on “Current Affairs” and “Governance”. Option (b) is wrong as it is an extreme statement. Option (c) is clearly wrong as there is no direct link with UPI. Option (d) is wrong as direct benefits transfer is not (exclusively) dependent on UPI. Hence, (a) is correct. This question checked your common sense, and will be rated EASY! (Relevant link is: http://www.npci.org.in/UPI_Background.aspx )


19. The terms ‘Event Horizon’, ‘Singularity’ ‘String Theory’ and ‘Standard Model’ are sometimes seen in the news in the context of

  1. Observation and understanding of the Universe
  2. Study of the solar and the lunar eclipses
  3. Placing satellites in the orbit of the Earth
  4. Origin and evolution of living organisms on the Earth.

Sol. Ans (a) This was an easy one for those who do not restrict themselves to the “Syllabus wordings” but go beyond that. A question based on “Geography” and “Current Affairs”. In the very first lecture in Geography module (can view video here : https://www.youtube.com/watch?v=sEqk5wHyuF4 ) we explained what a “Singularity” is. That term alone is enough to answer this question, and options (b), (c) and (d) are clearly wrong. Hence (a) is the correct answer. Remember that a lot has been appearing in newspapers on related developments (in context of Stephen Hawking’s discussions, telescopic studies etc.) Even a non-Physics student can easily answer this.(A relevant link is: http://www.physicsoftheuniverse.com/topics_blackholes_singularities.html )


20. With reference to agriculture in India, how can the technique of ‘genome sequencing’ often seen in the news, be used in the immediate future?

  1. Genome sequencing can be used to identify genetic markers for disease resistance and drought tolerance in various crop plants.
  2. This technique helps in reducing the time required to develop new varieties of crop plants.
  3. It can be used to decipher the host-pathogen relationships in crops.

Select the correct answer using the code given below:

  1. 1 only
  2. 2 and 3 only
  3. 1 and 3 only
  4. 1, 2 and 3

Sol. Ans (d) A question based on “Science and Technology” and “Agriculture”. The problems facing Indian agriculture, and the application of genetic science to enhance productivity are hot issues. (Whole) genome sequencing is the process of determining the complete DNA sequence of an organism's genome at a single time. It has many applications, including identifying genetic markers for disease resistance, developing new varieties of crops (which are more productive) and finding the host-pathogen relationships in crops. Hence (d) is the correct answer. (Reference link: http://www.genomeindia.org/rice/structural/about.htm)


21. The main advantage of the parliamentary form of government is that

  1. the executive and legislature work independently.
  2. It provides continuity of policy and is more efficient.
  3. the executive remains responsible to the legislature.
  4. the head of the government cannot be changed without the election.

Sol. Ans (c) A question based on “Constitution of India”. Option (a) is wrong. Executive in India is not independent (like it is in America), but is subordinate to the legislature. Political executive (government) and administrative executive both are not independent. Continuity of policy is surely a good point, but regular transfers can undermine that too. Head of the government is the Prime Minister who can be changed without an election, hence (d) is wrong. So option (c) is correct. This issue has been discussed multiple times across our course modules in various ways.


22. In the context of India, which one of the following is the correct relationship between Rights and Duties?

  1. Rights are correlative with Duties.
  2. Rights are personal and hence independent of society and Duties.
  3. Rights, not Duties are important for the advancement of the personality of the citizen.
  4. Duties, not Rights are important for the stability of the State.

Sol. Ans (a) A question based on “Constitution of India”. Read all the options carefully. Option (b) can be eliminated as that is absurd. Option (c) is again wrong because rights in the absence of duties are pernicious. Option (d) is taking a one-way view of what rights and duties alone mean for the State. We should not forget that our rights are ultimately someone else’s duties. Hence, (d) is wrong. Option (a) is perfectly right.


23. The mind of the makers of the Constitution of India is reflected in which of the following?

  1. The Preamble
  2. The Fundamental Rights
  3. The Directive Principles of State Policy
  4. The Fundamental Duties

Sol. Ans (a) A question based on “Constitution of India”. Option (d) is wrong as Fundamental Duties were added to the constitution nearly 3 decades after it was formed. Option (c) is all about the direction the state should work towards (the DPSPs). Option (b) is about the guarantee to the citizens of India and it is the bedrock of the limits to State power. Option (a) best represents the “summary” of the constitution, its soul and spirit. Best answer is option (a). (Reference link : http://www.legalservicesindia.com/article/article/is-preamble-a-part-of-constitution-2003-1.html)


24. If you travel by road from Kohima to Kottayam, what is the minimum number of States within India through which you can travel, including the origin and the destination?

  1. 6
  2. 7
  3. 8
  4. 9

Sol. Ans (b) A question based on “Geography”. The seven States are Manipur, Assam, West Bengal, Orissa, Andhra Pradesh, Tamil Nadu and Kerela. Source : Google Maps. You need to know the Indian states layout and cities very well to solve this correctly. Kohima (the capital of Nagaland) and Kottayam (Kerala) are two extremes of India!


25. The Parliament of India exercises control over the functions of the Council of Ministers through

  1. Adjournment motion
  2. Question hour
  3. Supplementary questions

Select the correct answer using the code given below:

  1. 1 only
  2. 2 and 3 only
  3. 1 and 3 only
  4. 1, 2 and 3

Sol. Ans (d) A question based on “Constitution of India” and “Parliamentary processes”. Parliament members can ask questions which the Ministers are bound to give reply to. This can happen through the “Question Hour” (first hour of a sitting session of Lok Sabha when a Minister is obliged to answer, either orally or in writing) or the “Supplementary Questions” (starred questions getting a Minister’s reply, to which a supplementary question can again be asked). An adjournment motion is moved to discuss a matter of utter public importance (which may or may not be to the liking of the Council of Ministers), and the House proceedings will stop to first discuss the matter. Hence, 1, 2 and 3 are correct, and best answer is (d).


26. With reference to the Parliament of India, consider the following statement :

  1. A private member’s bill is a bill presented by a Member of Parliament who is not elected but only nominated by the President’s of India.
  2. Recently , a private member’s bill has been passed in the Parliament of India for the first time in its history.

Which of the statements given above is/are correct?

  1. 1 only
  2. 2 only
  3. Both 1 and 2
  4. Neither 1 nor 2

Sol. Ans (d) A question based on “Constitution of India” and “Parliamentary processes”. Members of parliament other than ministers are called “private members” and bills presented by them are known as private member's bills. So Statement 1 is wrong. Many private members’ bills have been passed by the Parliament so far (14 to the last count). So statement 2 is wrong. Hence, the best answer is (d).
(Reference link : http://164.100.47.132/LssNew/abstract/private_members.htm and http://www.prsindia.org/media/articles-citing-prs/only-14-private-members-bills-passed-since-independence-3759)


27. With reference to the difference between the culture of Rigvedic Aryans and Indus Valley people, which of the following statements is/are correct?

  1. Rigvedic Aryans used the cost of mail and helmet in warfare whereas the people of Indus Valley Civilization did not leave any evidence of using them.
  2. Rigvedic Aryans knew gold, silver and copper whereas Indus Valley people knew only copper and iron.
  3. Rigvedic Aryans had domesticated the horse whereas there is no evidence of Indus Valley people having been aware of this animal.

Select the correct answer using the codes given below :

  1. 1 only
  2. 2 and 3 only
  3. 1 and 3 only
  4. 1, 2 and 3 only

Sol. Ans (a) If you check http://archaeologyonline.net/artifacts/horse-debate you find claims made that there were indeed physical remains of the horse in Indus-Sarasvati sites. A.K. Sharma’s well-known identification of horse remains at Surkotada (in Katchchh) was endorsed by global authorities, confirming several of them to be “remnants of true horses,” and what is more, domesticated horses! A horse figurines did emerge at Mohenjo-Daro. Also, if you go through https://www.harappa.com/slide/ancient-indus-ornaments and
http://heritage.gov.pk/html_pages/indus_jewlery_1.htm gold jewellery was found! So, correct answer should be (a) as 2 and 3 are wrong, but 1 is right.


28. ‘Recognition of Prior Learning Scheme’ is sometimes mentioned in the news with reference to

  1. Certifying the skills acquired by construction workers through traditional channels.
  2. Enrolling the persons in Universities for distance learning programmers.
  3. Reserving some skilled jobs to rural and urban poor in some public sector undertakings.
  4. Certifying the skills acquired by trainees under the National Skill Development Programme.

Sol. Ans (a) A question based on “Government Schemes”. The ‘Recognition of Prior Learning’ scheme is underway across construction sites in five states to certify skills acquired by workers through traditional, non-formal learning channels. So correct answer is (a). This is part of the much hyped skilling mission of GoI.
(Relevant links : http://indianexpress.com/article/india/india-others/formal-badge-for-non-formal-skill/
and http://www.nsdcindia.org/sites/default/files/files/Draft%20Guidelines%20for%20PMKVY%20RPL.pdf )


29. From the ecological point of view, which one of the following assumes importance in being a good link between the Eastern Ghats and the Western Ghats?

  1. Sathyamangalam Tiger Reserve
  2. Nallamala Forest
  3. Nagarhole National Park
  4. Seshachalam Biosphere Reserve

Sol. Ans (a) A question from “Indian Geography”. The Nallamala Forest (option (b)) are a section of the Eastern Ghats in Andhra Pradesh and Telangana. The Sathyamangalam Tiger Reserve is a wildlife sanctuary which lies between the Western Ghats and the rest of the Eastern Ghats. Option (c) - Nagarhole National Park – is in Western Ghats. The option (d) refers to Eastern Ghats only. Hence (a) is the best answer. (Reference link : https://en.wikipedia.org/wiki/Sathyamangalam_Wildlife_Sanctuary)


30. One of the implications of equality in society is the absence of

  1. Privileges
  2. Restraints
  3. Competition
  4. Ideology

Sol. Ans (a) A question from “Society and Culture” and also “Constitution and Law”. If a society has “Restraints”, then it can be on weaker sections also, hence egalitarianism may never happen. Even in a society that is not unequal (or has equality), there can be competition between its people. Ideology can exist in an equal society : it can be an ideology of true communism or egalitarianism. But “privileges” cannot exist in an equal society as they imply a special status for a few. Hence (a) is the best answer.
(Reference link : https://en.wikipedia.org/wiki/Privilege_(social_inequality))


31. Consider the following statements in respect of Trade Related Analysis of Fauna and Flora in Commerce [TRAFFIC]:

  1. TRAFFIC is a bureau under United Nations Environment Programme [UNEP]
  2. The mission of TRAFFIC is to ensure that trade in wild plants and animals is not a threat to the conservation of nature.

Which of the above statements is/are correct?

  1. 1 only
  2. 2 only
  3. Both 1 and 2
  4. Neither 1 nor 2

Sol. Ans (b) A question from “Environment Ecology and Climate Change”. TRAFFIC is a bureau under World Wildlife Fund for Nature not UNEP so Statement 1 is WRONG. Statement 2 is correct, as the mission of TRAFFIC is to ensure that trade in wild plants and animals is not a threat to conservation of nature. Hence answer is (b). (Reference link : https://en.wikipedia.org/wiki/Traffic_(conservation_programme)


32. Which principle among the following was added to the Directive Principles of State Policy by the 42 nd Amendment to the Constitution?

  1. Equal pay for equal work for both men and women
  2. Participation of workers in the management of industries.
  3. Right to work , education and public assistance
  4. Securing living wage and human conditions of work to workers

Sol. Ans (b) A question from “Constitution of India”. The 42nd amendment added 4 Directive Principles: 1. To secure opportunities for healthy development of children (Article 39), 2. To promote equal justice and to provide free legal aid to the poor (Article 39 A), 3. To take steps to secure the participation of workers in the management of industries (Article 43 A), and 4. To protect and improve the environment and to safeguard forests and wild life (Article 48 A). Hence (b) is correct. Note that the 44th Amendment 1978, and the 97th amendment 2011 also added more DPSPs. (Reference link : https://en.wikipedia.org/wiki/Directive_Principles


33. Which one of the following statements is correct?

  1. Rights are claims of the State against the citizens.
  2. Rights are privileges which are incorporated in the Constitution of a State
  3. Rights are claims of the citizens against the State
  4. Rights are privileges of a few citizens against the State

Sol. Ans (c) A question from “Society and culture” and “Constitution and Law”. Rights allow citizens to demand something specific from the state. Hence (c) is the best answer. Option (a) is reverse of it, hence wrong. Rights are not privileges, hence (b) is wrong. Option (d) is absurd. Option (c) is the correct answer.


34. Which of the following gives ‘Global Gender Gap Index’ ranking to the countries of the world?

  1. World Economic Forum
  2. UN Human Rights Council
  3. UN Women
  4. World Health Organization

Sol. Ans (a) A question from “Global current affairs”. The Global Gender Gap Report – an index designed to measure gender equality – was first published in 2006 by the World Economic Forum. The 2016 report covers 144 major and emerging economies. Best answer is (a). (Reference link : http://reports.weforum.org/global-gender-gap-report-2016/


35. Which of the following statements is/are correct regarding Smart India Hackathon 2017?

  1. It is a centrally sponsored scheme for developing every city of our country into Smart Cities in a decade.
  2. It is an initiative to identify new digital technology innovations for solving the many problems faced by our country.
  3. It is a programme aimed at making all the financial transactions in our country completely digital in a decade.

Select the correct answer using the code given below :

  1. 1 and 3 only
  2. 2 only
  3. 3 only
  4. 2 and 3 only

Sol. Ans (b) A question from “Current Affairs” and “Government Schemes”. ‘Smart India Hackathon 2017’ was launched in collaboration with i4c, MyGov, Persistent Systems, NASSCOM and Rambhau Mhalgi Prabodhini. It aims at building Digital India and to engage our youth directly with nation building. The 2017 event was a 36 hrs non-stop digital product development competition during which teams of thousands of technology students will build innovative digital solutions for the problems posted by 29 different central govt. ministries/ departments. (Reference link : http://pib.nic.in/newsite/PrintRelease.aspx?relid=159825) Only 2 is correct, and best answer is (b).


36. Which of the following statements is/are correct regarding the Monetary Policy Committed [MPC]?

  1. It decides the RBI’s benchmark interest rates
  2. It is a 12-member body including the Governor of RBI and is reconstituted every year.
  3. It functions under the chairmanship of the Union Finance Minister.

Select the correct answer using the code given below:

  1. 1 only
  2. 1 and 2 only
  3. 3 only
  4. 2 and 3 only

Sol. Ans (a) A question from “Indian economy” and “Current Affairs”. In the middle of many horrible questions, this one can bring a smile to your faces! We know how the MPC (3 members from RBI, 3 outside but not from government) were at loggerheads with the Finance Ministry in June 2017. Hence, statements 2 and 3 are directly ruled out. Only 1 is correct, and answer is (a).
(Reference link : http://pib.nic.in/newsite/PrintRelease.aspx?relid=151264)


37. With reference to Manipuri Sankirtana, consider the following statements :

  1. It is a song and a dance performance.
  2. Cymbals are the only musical instruments used in the performance.
  3. It is performed to narrate the life and deeds of Lord Krishna.

Which of the statements given above is/are correct?

  1. 1, 2 and 3
  2. 1 and 3 only
  3. 2 and 3 only
  4. 1 only

Sol. Ans (b) A question from “Art and Culture”. Manipuri Sankirtana is a popular song and dance performance performed in Manipur and the neighbouring regions which involve narrating the life of Lord Krishna through music and dance. Many instruments are used in this performance like cymbals, dhols, harmoniums etc. Statement 2 is clearly wrong. So options (a) and (c) are gone. Correct answer is (b). (Reference link : https://en.wikipedia.org/wiki/Manipuri_dance#Music_and_instruments)


38. Who among the following was/were associated with the introduction of Ryotwari Settlement in India during the British rule?

  1. Lord Cornwallis
  2. Alexander Read
  3. Thomas Munro

Select the correct answer using the code given below:

  1. 1 only
  2. 1 and 3 only
  3. 2 and 3 only
  4. 1, 2 and 3

Sol. Ans (c) A question from “Modern Indian History”. It was tough due to one name – Alexander Read. But you could eliminate three options as 1 – Lord Cornwallis was not associated with it! Ryotwari system was one of the three prime methods of revenue collection in British India, prevalent in most of southern India (Madras Presidency). The system was devised by Capt. Alexander Read and Thomas (later Sir Thomas) Munro at the end of the 18th century and introduced by the latter when he was governor (1820–27) of Madras (now Chennai). The principle was the direct collection of the land revenue from each individual cultivator by government agents, and elimination of middlemen. Correct answer is (c).


39. In the context of solving pollution problems, what is/are the advantage/advantages of bioremediation technique?

  1. It is a technique for cleaning up pollution by enhancing the same biodegradation process that occurs in nature.
  2. Any contaminant with heavy metals such as cadmium and lead can be readily and completely treated by bioremediation using microorganisms.
  3. Genetic engineering can be used to create microorganisms specifically designed for bioremediation.

Select the correct answer using the code given below:

  1. 1 only
  2. 2 and 3 only
  3. 1 and 3 only
  4. 1, 2 and 3

Sol. Ans (c) A question from “Environment Ecology and Climate Change”. You can see that in statement 2, the use of the word “completely” is extreme, hence we avoid that. Statements 1 and 3 are correct. Correct answer is (c).
(Reference link: https://www.ncbi.nlm.nih.gov/pmc/articles/PMC5026719)


40. The Trade Disputes Act of 1929 provided for

  1. the participation of workers in the management of industries.
  2. arbitrary powers to the management to quell industrial disputes.
  3. an intervention by the British Court in the event of a trade dispute
  4. a system of tribunals and a ban on strikes

Sol. Ans (d) A question from “Constitution and Law”. A tough one indeed! The Trade Disputes Act, 1929 was codified for five years as an experimental measure. The main object of the Act was to make provisions for establishment of Courts of Inquiry and Boards of Conciliation with a view to investigate and settle trade disputes. The Act prohibited strikes or lock-outs without notice in public utility services; it also made any strike or lock-out illegal. Correct answer is (c).
(Reference link : http://shodhganga.inflibnet.ac.in/bitstream/10603/8113/12/12_chapter%203.pdf)


41. Local self-government can be best explained as an exercise in

  1. Federalism
  2. Democratic decentralisation
  3. Administrative delegation
  4. Direct democracy

Sol. Ans (b) A question from “Governance”. This question is the most basic, and relates to the definition of local self-government itself. In order to ensure that democracy is exercised at the lowest rung of the society, we have urban and rural local self-governments – democratic decentralisation. Correct answer is (b).


42. Consider the following statements :

With reference to the Constitution of India, the Directive Principles of State Policy constitute limitations upon

  1. Legislative function.
  2. Executive function

Which of the above statements is/are correct?

  1. 1 only
  2. 2 only
  3. Both 1 and 2
  4. Neither 1 nor 2

Sol. Ans (d) A question from “Constitution of India”. Remember that the Directive Principles of State Policy expect a lot from both the government and the administration, and want them to make newer laws to widen the welfare state concept. So, the correct answer will be (d). A strongly conceptual question, like most other. Good one!


43. The term ‘Digital Single Market Strategy’ seen in the news refers to

  1. ASEAN
  2. BRICS
  3. EU
  4. G20

Sol. Ans (c) A question from “Global current affairs”. Factual question. The EU had proposed tearing down regulatory walls online and moving from 28 national markets (now 27!) to a single one. It is an ambitious project which involves removing boundaries amongst EU and promoting cross- country selling of goods and services across Europe. The correct answer will be (c).
(Resource Link : https://ec.europa.eu/commission/priorities/digital-single-market_en)


44. At one of the places in India , if you stand on the seashore and watch the sea, you will find that the sea water recedes from the shore line a few kilometres and comes back to the shore, twice a day, and you can actually walk on the sea floor when the water recedes. This unique phenomenon is seen at

  1. Bhavanagar
  2. Bheemunipatnam
  3. Chandipur
  4. Nagapattinam

Sol. Ans (c) A question from “Geography”. Chandipur – the famous site (Chandipur-on-sea) is also the location of the Indian Army's Integrated Test Range (ITR), at Abdul Kalam Island, formerly Wheeler Island. Maybe that’s why this was asked! The beach is unique as the water recedes up to 5 kilometres during the ebb tide. Answer is (c). Those aspirants who live in Chandipur must have relished this question!
(Reference link : https://en.wikipedia.org/wiki/Chandipur,_Odisha)


45. With reference to the ‘Prohibition of Benami Property Transactions Act, 1988 [PBPT Act]’ consider the following statements :

  1. A property transaction is not treated as a benami transaction if the owner of the property is not aware of the transaction.
  2. Properties held benami are liable for consfiscation by the Government.
  3. The Act provides for three authorities for investigations but does not provide for any appellate mechanism.

Which of the above statements is/are correct?

  1. 1 only
  2. 2 only
  3. 1 and 3 only
  4. 2 and 3 only

Sol. Ans (b) A question from “Current Affairs” and “New laws”. This was a hot topic, post demonetization, and questions were indeed expected! Remember, the Appellate Mechanism exists (that was a giveaway). So statement 3 is wrong, hence options (c) and (d) are wrong. We know that 2 is correct (so many political announcements lately), hence only option (b) is our desired answer!
(Reference link : http://pib.nic.in/newsite/PrintRelease.aspx?relid=153085)


46. Due to some reasons, if there is a huge fall in the population of species of butterflies, what could be its likely consequence/consequences?

  1. Pollination of some plants could be adversely affected.
  2. There could be a drastic increase in the fungal infections of some cultivated plants.
  3. It could lead to a fall in the population of some species of wasps, spiders and birds.

Select the correct answer using the code given below:

  1. 1 only
  2. 2 and 3 only
  3. 1 and 3 only
  4. 1, 2 and 3

Sol. Ans (c) A tough and logical one, from “Environment Ecology and Climate Change”. Statement 1 is correct as butterflies are good pollinating agents. Statement 2 is incorrect as decrease in butterfly populations would decrease fungal infections. Why? Because butterflies cause spreading of fungal infections as they carry fungal spores with them everywhere. Statement 3 is correct as some species of wasps, spiders and birds feed on butterflies and caterpillars. Hence, best answer is (c). (Relevant link : https://en.wikipedia.org/wiki/Butterfly)


47. It is possible to produce algae based biofuels, but what is/are the likely limitations[s] of developing countries in promoting this industry?

  1. Production of algae based biofuels is possible in seas only and not on continents.
  2. Setting up and engineering the algae based biofuel production requires high level of expertise/technology until the construction is completed.
  3. Economically viable production necessitates the setting up of large scale facilities which may rise ecological and social concerns.

Select the correct answer using the code given below:

  1. 1 and 2 only
  2. 2 and 3 only
  3. 3 only
  4. 1, 2 and 3

Sol. Ans (b) Statement (1) is wrong as algae based biofuels can also be produced in open ponds, artificial containers etc. although it is not preferred now (the question talks about “not possible” hence wrong).
(relevant link:https://en.wikipedia.org/wiki/Algae_fuel#Algae_cultivation ) Statements 2 and 3 are correct. It is a relatively new technology which currently requires expertise until construction is completed. As the yield isn’t that beneficial, large set-ups will be necessary which will raise ecological and social concerns. Best answer is (b). (Reference link:http://articles.extension.org/pages/26600/algae-for-biofuel-production )


48. Which of the following are the objectives of ‘National Nutrition Mission’?

  1. To create awareness relating to malnutrition among pregnant women and lactating mothers.
  2. To reduce the incidence of anaemia among young children, adolescent girls and women.
  3. To promote the consumption of millets, coarse cereals and unpolished rice.
  4. To promote the consumption of poultry egg.

Select the correct answer using the code given below:

  1. 1 and 2 only
  2. 1, 2 and 3 only
  3. 1, 2 and 4 only
  4. 3 and 4 only

Sol. Ans (a) A question from “Government Schemes”. There is no mention of consuming poultry eggs or millets, rice etc. in National Nutrition Mission, so statements 3 and 4 are wrong. So only possible answer is (a). (Relevant link : http://pib.nic.in/newsite/PrintRelease.aspx?relid=108509 )


49. Consider the following statements :

  1. The Factories Act, 1881 was passed with a view to fix the wages of industrial workers and to allow the workers to form trade unions.
  2. N.M.Lokhande was a pioneer in organizing the labour movement in British India.

Which of the above statements is/are correct?

  1. 1 only
  2. 2 only
  3. Both 1 and 2
  4. Neither 1 nor 2

Sol. Ans (b) Statement 1 is wrong as there was no provision for fixing industrial wages in 1881, a time when the British Raj was making people work until death, in the factories and mines! Also, there was no question of ‘allowing’ trade union formation either, though N M Lokhande and other were striving hard for the same. Statement 2 is correct. Answer is (b).(Reference link : https://en.wikipedia.org/wiki/Factory_Acts and https://en.wikipedia.org/wiki/Narayan_Meghaji_Lokhande )


50. In the context of mitigating the impending global warming due to anthropogenic emissions of carbon dioxide, which of the following can be the potential sites for carbon sequestration?

  1. Abandoned and uneconomic coal seams
  2. Depleted oil and gas reservoirs
  3. Subterranean deep saline formations

Select the correct answer using the code given below:

  1. 1 and 2 only
  2. 3 only
  3. 1 and 3 only
  4. 1, 2 and 3

Sol. Ans (d) A question from “Environment Ecology and Climate Change”. Carbon sequestration is the process involved in carbon capture and the long-term storage of atmospheric carbon dioxide. It involves long-term storage of carbon dioxide or other forms of carbon to mitigate or defer global warming. Artificial processes have been devised to produce similar effects, including large-scale, artificial capture and sequestration of industrially produced CO2 using subsurface saline aquifers, reservoirs, ocean water, aging oil fields, or other carbon sinks. So, option (d) is our desired answer! (Reference link : https://sequestration.mit.edu/)


51. Consider the following statements:

  1. In the election for Lok Sabha or State Assembly, the winning candidate must get at least 50 percent of the votes polled, to be declared elected.
  2. According to the provisions laid down in the Constitution of India, in Lok Sabha, the Speaker's post goes to the majority party and the Deputy Speaker's to the Opposition.

Which of the statements given above is/are correct?

  1. 1 only
  2. 2 only
  3. Both 1 and 2
  4. Neither 1 nor 2

Sol. Ans (d) A question from “Constitution and Law” and “Indian Polity”. India follows the “First Past the Post” system of elections and winning more than 50% is not the criterion, but getting the most is. There is no constitutional provision regarding Deputy Speaker, but it is a convention that an opposition party gets the post. Hence, 1 and 2 are both wrong. So, option (d) is our desired answer!


52. Which of the following has/have occurred in India after its liberalization of economic policies in 1991?

  1. Share of agriculture in GDP increased enormously.
  2. Share of India's exports in world trade increased.
  3. FDI inflows increased.
  4. India's foreign exchange reserves increased enormously.

Select the correct answer using the codes given below :

  1. 1 and 4 only
  2. 2, 3 and 4 only
  3. 2 and 3 only
  4. 1, 2, 3 and 4

Sol. Ans (b) Solution : (b) A question from “Indian Economy” and “Current Affairs”. We know very well that agriculture’s share as a % of our GDP has dropped continuously. Hence, 1 is wrong. Hence, options (a) and (d) are wrong. Now just check statement 4 to ge the final answer. Our forex reserves indeed grew post economic liberalization due to many factors. Hence, options (b) is the desired answer. A very easy question.


53. What is the application of Somatic Cell Nuclear Transfer Technology?

  1. Production of biolarvicides
  2. Manufacture of biodegradable plastics
  3. Reproductive cloning of animals
  4. Production of organisms free of diseases

Sol. Ans (c) A question from “Science and Technology”. You just had to have read it once earlier! Somatic Cell Nuclear Transfer Technology is used for reproductive cloning of animals. Answer is (c). (Reference link : https://www.ncbi.nlm.nih.gov/pmc/articles/PMC3539358)


54. Consider the following statements:

  1. National Payments Corporation of India (NPCI) helps in promoting the financial inclusion in the country.
  2. NPCI has launched RuPay, a card payment scheme.

Which of the statements given above is/are correct?

  1. 1 only
  2. 2 only
  3. Both 1 and 2
  4. Neither 1 nor 2

Sol. Ans (c) A question from “Government Schemes” A tough one, as the “financial inclusion” part confused many. Its official site says : “The NPCI is an umbrella organization for all retail payments system in India. It was set up with the guidance and support of RBI and Indian Banks’ Association (IBA). The core objective was to consolidate and integrate the multiple systems with varying service levels into nation-wide uniform and standard business process for all retail payment systems. The other objective was to facilitate an affordable payment mechanism to benefit the common man across the country and help financial inclusion.” Hence, 1 is correct. Also, RuPay is a new card payment scheme launched by the NPCI, so statement 2 is correct. Hence, answer is option (c). (Reference link : http://www.npci.org.in/aboutus.aspx)


55. The term M-STrIPES' is sometimes seen in the news in the context of

  1. Captive breeding of Wild Fauna
  2. Maintenance of Tiger Reserves
  3. Indigenous Satellite Navigation System
  4. Security of National Highways

Sol. Ans (b) An easy abbreviation-based question from “Environment Ecology and Climate Change”. “MSTrIPES” stands for “Monitoring System for Tigers – Intensive Protection & Ecological Status”. Hence, answer is option (b). (Reference link : http://admin.indiaenvironmentportal.org.in/files/mstripes-ppt.pdf )


56. What is/are the most likely advantages of implementing 'Goods and Services Tax (GST)'?

  1. It will replace multiple taxes collected by multiple authorities and will thus create a single market in India.
  2. It will drastically reduce the 'Current Account Deficit' of India and will enable it to increase its foreign exchange reserves.
  3. It will enormously increase the growth and size of economy of India and will enable it to overtake China in the near future.

Select the correct answer using the code given below:

  1. 1 only
  2. 2 and 3 only
  3. 1 and 3 only
  4. 1, 2 and 3

Sol. Ans (a) A question from “Indian Economy” and “Taxation – GST”. An easy one, perhaps as most are upset or scared about GST launch so soon! Statements 2 and 3 are absurd as they make extreme claims, mostly hypothetical and futuristic. But 1 is definitely correct! Best answer is option (a). Here is a comprehensive article on GST for your reference – http://www.bodhibooster.com/2016/12/GST-India-Council-Taxation-Direct-Indirect-Reform-States-GSTN-GSTCouncil-demonetisation.html


57. 'Broad-based Trade and Investment Agreement (BTIA)' is sometimes seen in the news in the context of negotiations held between India and

  1. European Union
  2. Gulf Cooperation Council
  3. Organization for Economic Cooperation and Development
  4. Shanghai Cooperation Organization

Sol. Ans (b) A direct question from “India’s International Affairs” and “Global trade”. Since 2007, when the India-EU BTIA negotiations started, 16 rounds of negotiations have been held at the level of Chief Negotiators. The last round of negotiations was held in 2013 and thereafter negotiations had been suspended. (Reference link : http://pib.nic.in/newsite/PrintRelease.aspx?relid=147137)


58. Consider the following statements:

  1. India has ratified the Trade Facilitation Agreement (TFA) of WTO.
  2. TFA is a part of WTO's Bali Ministerial Package of 2013.
  3. TFA came into force in January 2016.

Which of the statements given above is/are correct?

  1. 1 and 2 only
  2. 1 and 3 only
  3. 2 and 3 only
  4. 1, 2 and 3

Sol. Ans (a) A question from “Global current affairs”. Perhaps one of the easiest questions, as the TFA came into effect from February 2017 itself! It was widely covered by us in all sessions. So statement 3 is wrong, hence options (b), (c) and (d) are wrong! Hence, (a) is the answer required! (Reference link : https://www.wto.org/english/news_e/news17_e/fac_31jan17_e.htm)


59. What is the importance of developing Chabahar Port by India?

  1. India's trade with African countries will enormously increase.
  2. India's relations with oil-producing Arab countries will be strengthened.
  3. India will not depend on Pakistan for access to Afghanistan and Central Asia.
  4. Pakistan will facilitate and protect the installation of a gas pipeline between Iraq and India.

Sol. Ans (c) Another question from “India’s international affairs” and “Global current affairs”. Again, a sitter if only you followed this news carefully. In context of CPEC and Gwadar deep sea port, and also Iran’s objections to slow movement from India on Chabahar, you must know that options (a) and (b) are irrelevant to it. They are continuing even now. Option (d) is going against the grain of this issue. Best answer is (c), as Chabahar port allows an entry into Central Asia and Afghanistan to India, though with SCO membership that too may slowly become less of an issue. Best answer is (c). (Reference link : http://saar.bodhibooster.com/2017/04/India-INSTC-OBOR-Russia-Lapis-Lazuli-Chabahar.html)


60. In India, it is legally mandatory for which of the following to report on cyber security incidents?

  1. Service providers
  2. Data centres
  3. Body corporate

Select the correct answer using the code given below:

  1. 1 only
  2. 1 and 2 only
  3. 3 only
  4. 1, 2 and 3

Sol. Ans (d) In light of the cyber security concerns, ATM hacking incidents, and ransomware, this was an expected question from “Science and Technology” and “Governance”. The Information Technology Rules 2013 (The Indian Computer Emergency Response Team and Manner of Performing Functions and Duties) called the CERT Rules also impose an obligation on service providers, intermediaries, data centres and body corporates to report cyber incidents within a reasonable time so that CERT-In may have scope for timely action. Answer is (d). (Relevant link : https://cis-india.org/internet-governance/blog/incident-response-requirements-in-indian-law


61. Right to vote and to be elected in India is a

  1. Fundamental Right
  2. Natural Right
  3. Constitutional Right
  4. Legal Right

Sol. Ans (c) One of the most tricky questions! Multiple judicial pronouncements over the decades have culminated in a 2015 SC case called “Rajbala & Ors vs State Of Haryana & Ors on 10 December, 2015” in a bench of Justices Chelameswar, and Abhay Manohar Sapre which concluded that “In the light of aforementioned two authoritative pronouncements, we are of the considered opinion that both the rights namely "Right to Vote" and "Right to Contest" are constitutional rights of the citizen”. The tortuous history of this issue – (1) Article 326 of the Constitution provides for adult suffrage, but does not mention “the right to vote”, (2) The Sixty-first Amendment' of the Constitution of India, lowered the voting age from 21 years to 18 years, (3) SC held that rights that are not explicitly set out in the Constitution, such as the right to privacy, have routinely been impliedly read into the text, and the SC also said that “…but that treatment did not come to “the right to vote” and it was not always upheld as an inalienable constitutional right”, (4) Then the SC says “…It is disconcerting that the court still does not clearly acknowledge a constitutional right to vote. Participation in the electoral process is often seen as a gateway right, or a ‘right of rights’”, (5) This SC bench also spoke of “The Supreme Court, in its hearings in the N.P. Ponnuswami v. Returning Officer, Namakkal Constituency, Namakkal, Salem, AIR 1952 SC 64 and Jyoti Basu & Others v. Debi Ghosal & Others, (1982) 1 SCC 691, recorded the categoric conclusion that the “right to vote” if not a fundamental right is certainly a “constitutional right” and “it is not very accurate to describe it as a statutory right, pure and simple, and it said that the freedom of voting as distinct from right to vote is a facet of the fundamental right enshrined in Article 19(1)(a)”, (f) It also said that “Following the PUCL case, one of the SC judges held in Desiya Murpokku Dravida Kazhagam (DMDK) & Another v. Election Commission of India, (2012) 7 SCC 340: “…… every citizen of this country has a constitutional right both to elect and also be elected to any one of the legislative bodies created by the Constitution …….”. This should settle this question (of Prelims 2017) too! Best answer is (c). (Reference link : http://www.bodhibooster.com/p/legalreferences.html)


62. What is the purpose of 'evolved Laser Interferometer Space Antenna (eLISA)' project?

  1. To detect neutrinos
  2. To detect gravitational waves
  3. To detect the effectiveness of missile defence system
  4. To study the effect of solar flares on our communication systems

Sol. Ans (b) A question from “Current Affairs”. Evolved Laser Interferometer Space Antenna (eLISA) has been developed to detect gravitational waves, tiny ripples in the fabric of space-time — from astronomical sources. LISA would be the first dedicated space-based gravitational wave detector. It aims to measure gravitational waves directly by using laser interferometry. Best answer is (b). (Reference link : https://en.wikipedia.org/wiki/Laser_Interferometer_Space_Antenna)


63. What is the purpose of Vidyanjali Yojana'?

  1. To enable the famous foreign educational institutions to open their campuses in India.
  2. To increase the quality of education provided in government schools by taking help from the private sector and the community.
  3. To encourage voluntary monetary contributions from private individuals and organizations so as to improve the infrastructure facilities for primary and secondary schools.

Select the correct answer using the code given below :

  1. 2 only
  2. 3 only
  3. 1 and 2 only
  4. 2 and 3 only

Sol. Ans (a) A question from “Government Schemes”. Vidyanjali is an initiative to enhance community and private sector involvement in Government run elementary schools across the country under the overall aegis of the Sarva Shiksha Abhiyan. Through this initiative people from the Indian Diaspora, retired teachers, retired government officials including retired defense personnel, retired professionals and women who are home makers can volunteer at a school that requests for one. So statement 1 is ruled out, and 3 too is wrong as the scheme nowhere focuses on financial help and donations from private organizations. Answer is (a). (Reference link : http://vidyanjali.mygov.in/index.php/frontend/guideline )


64. What is the aim of the programme `Unnat Bharat Abhiyan'?

  1. Achieving 100% literacy by promoting collaboration between voluntary organizations and government's education system and local communities.
  2. Connecting institutions of higher education with local communities to address development challenges through appropriate technologies.
  3. Strengthening India's scientific research institutions in order to make India a scientific and technological power.
  4. Developing human capital by allocating special funds for health care and education of rural and urban poor, and organizing skill development programmes and vocational training for them.

Sol. Ans (b) A question from “Government Schemes”. This aims to involve professional and other higher educational institutions of the country in the process of indigenous development of self-sufficient and sustainable village clusters in tune with the notion of 'Gram Swaraj' propounded by Mahatma Gandhi. It proposes to reorient the ethos, the academic curricula and research programs in professional institutions to make these more compatible with the national needs including those of rural areas, etc. Clearly option (b) is the correct answer. (Reference link : http://unnat.iitd.ac.in/index.php/en/)


65. Consider the following statements :

  1. The Election Commission of India is a five-member body.
  2. Union Ministry of Home Affairs decides the election schedule for the conduct of both general elections and bye-elections.
  3. Election Commission resolves the disputes relating to splits/mergers of recognised political parties.

Which of the statements given above is/are correct ?

  1. 1 and 2 only
  2. 2 only
  3. 2 and 3 only
  4. 3 only

Sol. Ans (d) A question from “Indian Polity” and “Governance and Institutions”. The Election Commission is a 3 member body, and decides all election schedules. The Home Ministry helps arrange the paramilitary and CAPF troops (central forces). Hence, statements 1 and 2 are wrong, making options (a), (b), (c) wrong, hence (d) is the answer! Best answer is (d).
(Reference and download links : http://eci.nic.in/eci_main1/the_setup.aspx#appointment-tenure,http://resources.bodhisaar.com/p/home.html and http://eci.nic.in/eci_main1/the_function.aspx#schedulingelec)


66. In India, if a species of tortoise is declared protected under Schedule I of the Wildlife (Protection) Act, 1972, what does it imply ?

  1. It enjoys the same level of protection as the tiger.
  2. It no longer exists in the wild, a few individuals are under captive protection; and now it is impossible to prevent its extinction.
  3. It is endemic to a particular region of India.
  4. Both (b) and (c) stated above are correct in this context.

Sol. Ans (a) As the tiger is a species listed in the Schedule 1 of the Indian Wildlife Act 1972, it means if a species of tortoise is declared protected under schedule 1, it will enjoy same level of protection the tiger does. Option (b) is wrong for obvious reasons, (c) is not related to wildlife act and (d) is wrong as (b) is wrong. Hence answer is (a). (Reference link : http://www.bodhibooster.com/p/legalreferences.html#post-page-number-2 )


67. In India, Judicial Review implies

  1. the power of the Judiciary to pronounce upon the constitutionality of laws and executive orders.
  2. the power of the Judiciary to question the wisdom of the laws enacted by the Legislatures.
  3. the power of the Judiciary to review all the legislative enactments before they are assented to by the President.
  4. the power of the Judiciary to review its own judgements given earlier in similar or different cases.

Sol. Ans (a) Another one from “Indian Polity” and “Constitution and Law”. Judicial Review is the strongest element in maintaining “checks and balances” in Indian system. It refers to the constitutional power given to judiciary to check if laws made, or executive actions taken, are within the constitutional boundaries or not. Option (b) uses the wrong words. Option (c) is absurd and factually wrong. Option (d) is not what this system is primarily about. Hence (a) is the answer. (Reference link : http://www.yourarticlelibrary.com/essay/judicial-review-in-india-concept-provisions-amendments-and-other-details/24911/ )


68. With reference to Indian freedom struggle, consider the following events :

  1. Mutiny in Royal Indian Navy
  2. Quit India Movement launched
  3. Second Round Table Conference

What is the correct chronological sequence of the above events?

  1. 1-2-3
  2. 2-1-3
  3. 3-2-1
  4. 3-1-2

Sol. Ans (c) A simple one from “Modern Indian History”. If you knew that the RIN revolt (statement 1) was the final nail in the British Raj’s coffin in India, you’d know that 3-2-1 is the answer, hence option (c). Now, the problem is (a) has 1-2-3 as the sequence. But we assume that “chronological sequence” will mean time going forward, not backward, hence option (c).


69. Consider the following statements :

  1. Tax revenue as a percent of GDP of India has steadily increased in the last decade.
  2. Fiscal deficit as a percent of GDP of India has steadily increased in the last decade.

Which of the statements given above is/are correct ?

  1. 1 only
  2. 2 only
  3. Both 1 and 2
  4. Neither 1 nor 2

Sol. Ans (d) A direct one from “Indian Economy”. Tax Revenue of India has not grown continuously over the past decade but has either remained stable, or dropped in some years. That is the bane of our government revenue system, leading to many moves, including the urgency in GST implementation etc.Fiscal deficit luckily has been managed well, starting 2003 due to the FRBM framework (it slipped post 2007 GFC, but that’s another story). Hence, both statements 1 and 2 are wrong. Correct answer is (d). (Reference links and images : http://saar.bodhibooster.com/2017/02/India-tax-to-gdp-ratio-GST-direct-indirect-taxes.html and http://saar.bodhibooster.com/2017/02/India-tax-to-gdp-ratio-GST-direct-indirect-taxes.html)


70. Recently there was a proposal to translocate some of the lions from their natural habitat in Gujarat to which one of the following sites ?

  1. Corbett National Park
  2. Kuno Palpur Wildlife Sanctuary
  3. Mudumalai Wildlife Sanctuary
  4. Sariska National Park

Sol. Ans (b) A direct one from “Current Affairs”. The group called Wildlife Institute of India (Dehradun) had proposed that 40 lions from Gujarat be shifted to Madhya Pradesh’s KunoPalpur Wildlife sanctuary, as per SC order not executed since 3 years! Correct answer is (b). (Reference links and images : http://timesofindia.indiatimes.com/home/environment/flora-fauna/No-sign-of-Gir-transfer-MP-seeks-to-shift-zoo-lions-to-Kuno/articleshow/45379624.cms)


71. Which of the following are not necessarily the consequences of the proclamation of the President's rule in a State?

  1. Dissolution of the State Legislative Assembly
  2. Removal of the Council of Ministers in the State
  3. Dissolution of the local bodies

Select the correct answer using the code given below:

  1. 1 and 2 only
  2. 1 and 3 only
  3. 2 and 3 only
  4. 1, 2 and 3

Sol. Ans (b) A direct one from “Indian Polity”. When President’s rule is imposed in a state, the Council of Ministers is removed immediately. But when the President’s Rule is removed, the Assembly may come reconvene for a session again. Local Bodies remain unaffected from all this. So, statements 1 and 3 are correct, and answer is option (b)


72. Which of the following are envisaged by the Right against Exploitation in the Constitution of India?

  1. Prohibition of traffic in human beings and forced labour
  2. Abolition of untouchability
  3. Protection of the interests of minorities
  4. Prohibition of employment of children in factories and mines

Select the correct answer using the code given below:

  1. 1, 2 and 4 only
  2. 2, 3 and 4 only
  3. 1 and 4 only
  4. 1, 2, 3 and 4

Sol. Ans (c) A direct one from “Indian Polity”. If you read the fundamental rights chapter properly, you’d surely know that untouchability (Article 17 – Right to Equality) and minority interests (Article 29 – Cultural and Educational Rights) are covered by other fundamental rights, and not Right against Exploitation (Articles 23 and 24 – which covers human trafficking and child labour). Best answer is (c). (Reference link : http://indiacode.nic.in/coiweb/coifiles/p03.htm)


73. Which of the following is geographically closest to Great Nicobar ?

  1. Sumatra
  2. Borneo
  3. Java
  4. Sri Lanka

Sol. Ans (a) A direct one from “Geography”, which expected you to know the relative placement of regions on a map (you surely cannot remember all “distances” but can memorise the general broad placement of regions). The Great Nicobar island is the southernmost island in Andaman and Nicobar chain, and lies right next to Sumatra island of Indonesia. Java comes after that, and Borneo much beyond (and north) to that. Sri Lanka is too far off! Best answer is (a). (Reference link : http://www.bodhibooster.com/p/geographyinputs.html)


74. Out of the following statements, choose the one that brings out the principle underlying the Cabinet form of Government:

  1. An arrangement for minimizing the criticism against the Government whose responsibilities are complex and hard to carry out to the satisfaction of all.
  2. A mechanism for speeding up the activities of the Government whose responsibilities are increasing day by day.
  3. A mechanism of parliamentary democracy for ensuring collective responsibility of the Government to the people.
  4. A device for strengthening the hands of the head of the Government whose hold over the people is in a state of decline.

Sol. Ans (c) In light of what’s happening in India today, this is an interesting question! Options (a) and (d) are not how it was conceptualized originally! Hence, wrong. Option (b) looks correct, but the technical idea is covered in option (c). Entire council of ministers (Cabinet form of government) has to resign if any one member’s bill in Lok Sabha fails to get approval. Do not get confused between “Council of Ministers” versus “Cabinet (as a part of CoM)” because the question mentioned “Cabinet form of Government” which means “Our present system of CoM based government of collective responsibility”. Best answer is (c). (Reference link : https://en.wikipedia.org/wiki/Union_Council_of_Ministers)


75. Which one of the following is not a feature of Indian federalism?

  1. There is an independent judiciary in India.
  2. Powers have been clearly divided between the Centre and the States.
  3. The federating units have been given unequal representation in the Rajya Sabha.
  4. It is the result of an agreement among the federating units.

Sol. Ans (d) Another one from “Indian Polity” and “Current Affairs”, with a clear political tone to it, as allegations by some States against the present government of excessive concentration of power at Centre are common. It is obvious that Indian judiciary is almost independent. That is strictly speaking not a defining part of “federalism”, but of “checks and balances” in the system. However, since (d) is completely wrong (factually) as our states (federating units) did not even form finally at the time the constitution and quasi-federal structure was created. That had happened in USA in the 18th century, not here! Hence, (d) is wrong. That is our answer. (Reference link : an interesting insight comes from the definition of various States in our Constitution’s First Schedule, here, http://www.lawmin.nic.in/coi/FIRST-SCHEDULE.pdf)


76. Which one of the following was a very important seaport in the Kakatiya kingdom?

  1. Kakinada
  2. Motupalli
  3. Machilipatnam (Masulipatnam)
  4. Nelluru

Sol. Ans (b / c) A question from “Ancient History”. Both the options (b) and (c) seem correct as both qualify to be “a very important seaport in the Kakatiya kingdom” if one checks multiple sources, old and contemporary. Answer is ambiguous. (Relevant link : Chakravarti 1991, https://en.wikipedia.org/wiki/Kakatiya_dynasty)


77. With reference to 'Global Climate Change Alliance', which of the following statements is/are correct?

  1. It is an initiative of the European Union.
  2. It provides technical and financial support to targeted developing countries to integrate climate change into their development policies and budgets.
  3. It is coordinated by World Resources Institute (WRI) and World Business Council for Sustainable Development (WBCSD).

Select the correct answer using the code given below:

  1. 1 and 2 only
  2. 3 only
  3. 2 and 3 only
  4. 1, 2 and 3

Sol. Ans (a) A question from “Environment Ecology and Climate Change”. The Global Climate Change Alliance (GCCA) is an initiative of the European Union (EU) to strengthen dialogue and cooperation on climate change with developing countries most vulnerable to climate change. The initiative was launched in 2007 and is coordinated by the European Commission (EC). Both 1 and 2 are correct, and answer is (a). (Relevant link : http://www.gcca.eu/about-the-gcca/what-is-the-gcca)


78. With reference to the religious history of India, consider the following statements:

  1. Sautrantika and Sammitiya were the sects of Jainism.
  2. Sarvastivadin held that the constituents of phenomena were not wholly momentary, but existed forever in a latent form.

Which of the statements given above is/are correct?

  1. 1 only
  2. 2 only
  3. Both 1 and 2
  4. Neither 1 nor 2

Sol. Ans (b) A question from “Religion” and “Ancient History”. The Sautrāntika were an early Buddhist school generally believed to be descended from the Sthavira nikāya by way of their immediate parent school, the Sarvāstivādins. So statement 1 is wrong. The Sarvāstivāda were an early school of Buddhism that held to the existence of all dharma in the past, present and future, which can be interpreted as statement 2, hence 2 is correct. So our answer is (b).
(Relevant links : https://en.wikipedia.org/wiki/Sarvastivada#Early_history and https://en.wikipedia.org/wiki/Sautrāntika )


79. Mediterranean Sea is a border of which of the following countries?

  1. Jordan
  2. Iraq
  3. Lebanon
  4. Syria

Select the correct answer using the code given below:

  1. 1, 2 and 3 only
  2. 2 and 3 only
  3. 3 and 4 only
  4. 1, 3 and 4 only

Sol. Ans (c) A question from “Geography” and “Current Affairs”. Mediterranean Sea (incubator of western civilisation and a "sea between the lands") stretches from the Atlantic Ocean on the west to Asia on the east and separates Europe from Africa. It forms the border for Lebanon and Syria, but not Jordan or Iraq. The trend in past few years has been to ask geography questions in context of hot current affairs – and the Middle East and Iraq/Syria surely qualify for that! Answer is (c). (Relevant link : http://www.bodhibooster.com/p/geographyinputs.html)


80. With reference to 'National Investment and Infrastructure Fund', which of the following statements is/are correct?

  1. It is an organ of NITI Aayog.
  2. It has a corpus of Rs. 4, 00,000 crore at present.

Select the correct answer using the code given below:

  1. 1 only
  2. 2 only
  3. Both 1 and 2
  4. Neither 1 nor 2

Sol. Ans (d) A question from “Governance” and “Government Schemes”. Statement 2 is incorrect because the corpus it has is around Rs.40,000 crore not Rs.4,00,000 crore. That was a fine distinction, a huge one actually, and you should have read the figures carefully. Statement 1 is incorrect because is it a fund created by the government of India, and the NITI AAYOG is nowhere related to it. Answer is (d).


81. The Global Infrastructure Facility is a/an

  1. ASEAN initiative to upgrade infrastructure in Asia and financed by credit from the Asian Development Bank.
  2. World Bank collaboration that facilitates the preparation and structuring of complex infrastructure Public-Private Partnerships (PPPs) to enable mobilization of private sector and institutional investor capital.
  3. Collaboration among the major banks of the world working with the OECD and focused on expanding the set of infrastructure projects that have the potential to mobilize private investment.
  4. UNCTAD funded initiative that seeks to finance and facilitate infrastructure development in the world.

Sol. Ans (b) A question from “Infrastructure”. The Global Infrastructure Facility (GIF) is a global, open platform that facilitates the preparation and structuring of complex infrastructure public-private partnerships (PPPs), to enable mobilization of private sector and institutional investor capital. It is in collaboration with World Bank. So clearly the correct answer is (b). (Relevant link : http://www.globalinfrafacility.org/what-is-the-gif)


82. For election to the Lok Sabha, a nomination paper can be filed by

  1. anyone residing in India.
  2. a resident of the constituency from which the election is to be contested.
  3. any citizen of India whose name appears in the electoral roll of a constituency.
  4. any citizen of India.

Sol. Ans (c) Another question from “Polity” and “Constitution and law”. Option (a) is wrong as there can be aliens too. Article 84 (Part V—The Union) of Indian Constitution sets qualifications for being a member of Lok Sabha, which tell us that (a) and (d) are wrong or incomplete. Now the option (b) isn’t a necessary condition to be fulfilled, the best answer here remains (c). “He / She should have his/her name in the electoral rolls in any part of the country.” The ECI states that – “Q 4. I am registered as a voter in Delhi. Can I contest election to Lok Sabha from Haryana or Maharashtra, or Orissa? Ans. Yes. If you are a registered voter in Delhi, you can contest an election to Lok Sabha from any constituency in the country except Assam, Lakshadweep and Sikkim, as per Section 4 (c), 4 (cc) and 4 (ccc) of the R. P. Act, 1951.” Answer is (c). (Relevant link : http://www.bodhibooster.com/p/legalreferences.html#post-page-number-3)


83. Consider the following statements:

  1. In India, the Himalayas are spread over five States only.
  2. Western Ghats are spread over five States only.
  3. Pulicat Lake is spread over two States only.

Which of the statements given above is/are correct?

  1. 1 and 2 only
  2. 3 only
  3. 2 and 3 only
  4. 1 and 3 only

Sol. Ans (b) A tough question, that looks simple, from “Physical Geography”. The Indian Himalayan Region (IHR) is the section of the Himalayas within India, spanning multiple states – Jammu & Kashmir (41%), Himachal Pradesh (10%), Uttarakhand (10%), Sikkim (1%), West Bengal hills (0.59%), Meghalaya (4%), Assam hills (2.8%), Tripura (1.9%), Mizoram (3.95%), Manipur (4.18%), Nagaland (3%), and Arunachal Pradesh (15.6%). So 1 is wrong. Western Ghats are spread over 6 states, Gujarat, Maharashta, Goa, Karnataka, Kerela and Tamil Nadu. So 2 is wrong. Pulicat Lake is the second largest brackish water lake, after Chilika, and straddles the border of Andhra Pradesh (40%) and Tamil Nadu (60%), on the Coromandal Coast in South India. Only possible solution is (b). (Relevant link : http://www.bodhibooster.com/p/geographyinputs.html)


84. Biological Oxygen Demand (BOD) is a standard criterion for

  1. Measuring oxygen levels in blood
  2. Computing oxygen levels in forest ecosystems
  3. Pollution assay in aquatic ecosystems
  4. Assessing oxygen levels in high altitude regions

Sol. Ans (c) A straight and simple question from “Environment, Ecology and Climate Change” (simple because the options could be easily eliminated). The BOD (Biological Oxygen Demand) is the measure of oxygen required by bacteria and other unwanted species to oxidize organic matter in an aquatic ecosystem(aerobic), so it is a standard criterion for pollution assay in aquatic ecosystem. More BOD means more polluted water source. Answer is (c). (Relevant link : https://realtechwater.com/biochemical-oxygen-demand


85. With reference to the role of UN-Habitat in the United Nations programme working towards a better urban future, which of the statements is/are correct?

  1. UN-Habitat has been mandated by the United Nations General Assembly to promote socially and environmentally sustainable towns and cities to provide adequate shelter for all.
  2. Its partners are either governments or local urban authorities only.
  3. UN-Habitat contributes to the overall objective of the United Nations system to reduce poverty and to promote access to safe drinking water and basic sanitation.

Select the correct answer using the code given below:

  1. 1, 2 and 3
  2. 1 and 3 only
  3. 2 and 3 only
  4. 1 only

Sol. Ans (b) A question from “International Organisations”. The United Nations Human Settlements Programme (UN–Habitat) is the United Nations agency for human settlements and sustainable urban development. The twin goals of the Habitat Agenda are adequate shelter for all and the development of sustainable human settlements in an urbanizing world. The mandate of UN-Habitat is further derived from other internationally agreed development goals, in particular the target on water and sanitation of the Plan of Implementation of the World Summit on Sustainable Development, which sought to halve, by the year 2015, the proportion of people without sustainable access to safe drinking water and basic sanitation. So 1 and 3 are correct. Statement 2 is incorrect as there is no such talk of partnering with either governments or local urbans authorities only. Hence, answer is (b). (Relevant link : https://unhabitat.org/about-us/un-habitat-at-a-glance/)


86. With reference to 'National Skills Qualification Framework (NSQF)', which of the statements given below is/are correct?

  1. Under NSQF, a learner can acquire the certification for competency only through formal learning.
  2. An outcome expected from the implementation of NSQF is the mobility between vocational and general education.

Select the correct answer using the code given below:

  1. 1 only
  2. 2 only
  3. Both 1 and 2
  4. Neither 1 nor 2

Sol. Ans (b) One more question from “Government Schemes”. The National Skills Qualifications Framework (NSQF) is a competency-based framework that organizes all qualifications according to a series of levels of knowledge, skills and aptitude. Under NSQF, the learner can acquire the certification for competency needed at any level through formal, non-formal or informal learning. So 1 is wrong (use of “only”). Mobility between vocational and general education by alignment of degrees with NSQF is fluid. So, clearly statement 1 is incorrect and 2 is correct. Answer is (b). (Relevant link : http://www.skilldevelopment.gov.in/nsqf.html )


87. In the context of Indian history, the principle of `Dyarchy (diarchy)' refers to

  1. Division of the central legislature into two houses.
  2. Introduction of double government i.e., Central and State governments.
  3. Having two sets of rulers; one in London and another in Delhi.
  4. Division of the subjects delegated to the provinces into two categories.

Sol. Ans (d) Another question from the “Indian History” domain. Dyarchy was a system of double government introduced by the Government of India Act (1919) for the provinces of British India. It was introduced as a constitutional reform by Edwin Samuel Montagu (secretary of state for India, 1917–22) and Lord Chelmsford (viceroy of India, 1916–21). In this, the multiple subjects of administration were divided between the councillors and the ministers, and called reserved and transferred subjects, respectively. Do not confuse “diarchy” with “bicameralism” (two houses of a legislature) or with “federalism” (Centre and State governments). Best answer is (d).
(Reference link : https://en.wikipedia.org/wiki/Government_of_India_Act,_1919)


88. Consider the following in respect of 'National Career Service':

  1. National Career Service is an initiative of the Department of Personnel and Training, Government of India.
  2. National Career Service has been launched in a Mission Mode to improve the employment opportunities to uneducated youth of the country.

Which of the above statements is/are correct?

  1. 1 only
  2. 2 only
  3. Both 1 and 2
  4. Neither 1 nor 2

Sol. Ans (b) Another question from “Government Schemes” based on skills development. The NCS is an initiative of the Ministry of Labour and Employment so Statement 1 is wrong. ‘A National ICT based portal is developed primarily to connect the opportunities with the aspirations of youth. This portal facilitates registration of job seekers, job providers, skill providers, career counsellors, etc.’ so Statement 2 is correct (skill providers means uneducated youth also involved). Best answer is (b). (Relevant link : https://www.ncs.gov.in/pages/about-us.aspx)


89. Which of the following statements best k describes the term 'Scheme for Sustainable Structuring of Stressed Assets (S4A)', recently seen in the news?

  1. It is a procedure for considering ecological costs of developmental schemes formulated by the Government.
  2. It is a scheme of RBI for reworking the financial structure of big corporate entities facing genuine difficulties.
  3. It is a disinvestment plan of the Government regarding Central Public Sector Undertakings.
  4. It is an important provision in 'The Insolvency and Bankruptcy Code' recently implemented by the Government.

Sol. Ans (b) A much expected question on NPA, from the “Indian Economy” domain! The huge NPA problem facing public sector banks in India witnessed an “alphabetical soup” of schemes proposed over the past few years. The SSSSA (S4A) was one such scheme by the RBI, launched in June 2016, as part of its effort to resolve the NPA crisis. It failed to do so substantially, and finally the insolvency route is being taken in 2017. Answer is (b). (Reference link : https://www.rbi.org.in/scripts/BS_PressReleaseDisplay.aspx?prid=37210)


90. Consider the following statements:

  1. Climate and Clean Air Coalition (CCAC) to Reduce Short Lived Climate Pollutants is a unique initiative of G20 group of countries.
  2. The CCAC focuses on methane, black carbon and hydrofluorocarbons.

Which of the above statements is/are correct?

  1. 1 only
  2. 2 only
  3. Both 1 and 2
  4. Neither 1 nor 2

Sol. Ans (b) A straight and simple question from “Environment, Ecology and Climate Change”. The Climate and Clean Air Coalition to Reduce Short-Lived Climate Pollutants (CCAC) was launched by the United Nations Environment Programme (UNEP) and six countries—Bangladesh, Canada, Ghana, Mexico, Sweden, and the United States—on 16 February 2012. So Statement 1 is wrong. CCAC focuses on methane, carbon and hydrofluorocarbons. So 2 is correct. Hence answer is (b).
(Reference links :
https://en.wikipedia.org/wiki/Climate_and_Clean_Air_Coalition_to_Reduce_Short-Lived_Climate_Pollutants#Founding_Partners
, and
https://en.wikipedia.org/wiki/Climate_and_Clean_Air_Coalition_to_Reduce_Short-Lived_Climate_Pollutants )


91. With reference to 'Indian Ocean Dipole (IOD)' sometimes mentioned in the news while forecasting Indian monsoon, which of the following statements is/are correct?

  1. IOD phenomenon is characterised by a difference in sea surface temperature between tropical Western Indian Ocean and tropical Eastern Pacific Ocean.
  2. An IOD phenomenon can influence an El Nino's impact on the monsoon.

Select the correct answer using the code given below:

  1. 1 only
  2. 2 only
  3. Both 1 and 2
  4. Neither 1 nor 2

Sol. Ans (b) A from “Geography”. The Indian Ocean Dipole (IOD), also known as the Indian Niño, is an irregular oscillation of sea-surface temperatures in which the western Indian Ocean becomes alternately warmer and then colder than the eastern part of the ocean. So Statement 1 is incorrect, as it uses the term "Eastern Pacific Ocean" whereas it is Eastern Indian Ocean. So a play on the words! Monsoon in India is generally affected by the temperature between bay of Bangal in the east and the Arabian sea in the west. So statement 2 is correct.Hence, answer is (b). (Relevant link : https://en.wikipedia.org/wiki/Indian_Ocean_Dipole)


92. If you want to see gharials in their natural habitat, which one of the following is the best place to visit?

  1. Bhitarkanika Mangroves
  2. Chambal River
  3. Pulicat Lake
  4. Deepor Beel

Sol. Ans (b) A question from “Environment Ecology and Climate Change”. A pretty straight question, factual. Answer is (b). (Reference link : https://en.wikipedia.org/wiki/Chambal_River)


93. Consider the following in respect of Indian Ocean Naval Symposium (IONS):

  1. Inaugural IONS was held in India in 2015 under the chairmanship of the Indian Navy
  2. IONS is a voluntary initiative that seeks to increase maritime co-operation among navies of the littoral states of the Indian Ocean Region.

Which of the above statements is/are correct?

  1. 1 only
  2. 2 only
  3. Both 1 and 2
  4. Neither 1 nor 2

Sol. Ans (b) A question from “Defence and Military”, especially in light of the Chinese thrust into the IOR. The inaugural IONS-2008 was held in New Delhi, India on 14 Feb 2008. So statement 1 is wrong. The ‘Indian Ocean Naval Symposium’ (IONS) is a voluntary initiative that seeks to increase maritime co-operation among navies of the littoral states of the Indian Ocean Region by providing an open and inclusive forum for discussion of regionally relevant maritime issues. So 2 is correct. Hence, best answer is (b). (Reference link : http://www.bodhibooster.com/p/special-groupings.html)


94. The painting of Bodhisattva Padmapani is one of the most famous and oft-illustrated paintings at

  1. Ajanta
  2. Badami
  3. Bagh
  4. Ellora

Sol. Ans (a) A factual question from “Ancient Indian History”. The famous painting of the Bodhisattva Padmapani in Ajanata caves (a world Heritage site) is one of the best paintings remaining from the Pre historic era when Buddhism was at its peak. Best answer is (a). (Reference link : https://commons.wikimedia.org/wiki/File:Bodhisattva_Padmapani,_Ajanta,_cave_1,_India.jpg)


95. Consider the following pairs:

# Traditions Communities
1 Chaliha Sahib Festival - Sindhis
2 Nanda Raj Jaat Yatra - Gonds
3 Wari-Warkari - Santhals
  1. 1 only
  2. 2 and 3 only
  3. 1 and 3 only
  4. None of the above

Sol. Ans (a) A factual question from “Art and Culture”. The Chaliha is an important festival celebrated by the Sindhis wherein they pray to their God, Jhulelal. Hence statement 1 is correct. But statements 2 and 3 are factually wrong. The three-week-long Nanda Devi Raj Jat is a pilgrimage and festival of Uttarakhand in India, not of the Gonds. And, Varkari ("a pilgrim") is a sampradaya (religious movement) within the bhakti tradition, in Maharashtra and northern Karnataka (and not Santhals). So 3 is also wrong. Hence answer is (a).


96. Which of the following practices can help in water conservation in agriculture? ‎

  1. Reduced or zero tillage of the land ‎
  2. Applying gypsum before irrigating the field ‎
  3. Allowing crop residue to remain in the field ‎

Select the correct answer using the code given below : ‎

  1. 1 and 2 only‎
  2. 3 only ‎
  3. 1 and 3 only ‎
  4. 1, 2 and 3

Sol. Ans (d) A factual question from “Agriculture”. No-till farming (also called zero tillage or direct drilling) is a way of growing crops or pasture from year to year without disturbing the soil through tillage. No-till is an agricultural technique which increases the amount of water that infiltrates into the soil. So statement 1 is correct. Gypsum has been used for many years to improve aggregation and inhibit or overcome dispersion in sodic soils as it improves soil structure. It also improves water infiltration. Gypsum improves the ability of soil to drain and not become waterlogged due to a combination of high sodium, swelling clay and excess water. Gypsum also helps reduce runoff and erosion. So 2 is also correct. Statement 3 is correct for sure. Hence, best answer is (d).
(Relevant link : http://www.croplife.com/crop-inputs/micronutrients/the-role-of-gypsum-in-agriculture-5-key-benefits-you-should-know)


97. Consider the following statements : ‎

The nation-wide 'Soil Health Card Scheme' aims at

  1. expanding the cultivable area under irrigation.‎
  2. enabling the banks to assess the quantum of loans to be granted to farmers on the basis of soil quality. ‎
  3. checking the overuse of fertilizers in farmlands. ‎

Which of the above statements is/are correct?‎

  1. 1 and 2 only‎
  2. 3 only ‎
  3. 2 and 3 only ‎
  4. 1, 2 and 3‎

Sol. Ans (b) A factual question from “Government Schemes” and “Agriculture”. The soil health card scheme aims at promoting soil-test-based and balanced use of fertilisers to enable farmers realise higher yields at lower cost. So statement 3 is correct. The scheme does not talk about statements 1 and 2. Best answer is (b). (Relevant links : FAQ English - https://goo.gl/jfw5dn, FAQ Hindi - https://goo.gl/Lfu1X5)


98. Consider the following pairs:

# Commonly used / consumed materials Unwanted or controversial chemicals likely to be found in them
1 Lipstick Lead
2 Soft drinks Brominated vegetable oils
3 Chinese fast food Monosodium glutamate

Which of the pairs given above is/are correctly matched?

  1. 1 only
  2. 2 and 3 only
  3. 1 and 3 only
  4. 1, 2 and 3

Sol. Ans (d) A factual question from “Current Affairs”. Related to controversial issues recently! Answer is (d).
(Relevant links : http://www.safecosmetics.org/get-the-facts/regulations/us-laws/lead-in-lipstick,
https://en.wikipedia.org/wiki/Brominated_vegetable_oil)


99. Organic Light Emitting Diodes (OLEDs) are used to create digital display in many devices. What are the advantages of OLED displays over Liquid Crystal displays?

  1. OLED displays can be fabricated on flexible plastic substrates.
  2. Roll-up displays embedded in clothing can be made using OLEDs.‎
  3. Transparent displays are possible using OLEDs. ‎

Select the correct answer using the code given below‎

  1. 1 and 3 only‎
  2. 2 only ‎
  3. 1, 2 and 3‎
  4. None of the above statements is correct‎

Sol. Ans (c) A factual question from “Science and Technology”. OLED displayes are fabricated using flexible plastic substrates (FOLED). Clothing is already been embedded in roll up displayes. So statements 1 and 2 are correct, and only possible answer is (c).
(Relevant links : https://en.wikipedia.org/wiki/OLED, https://en.wikipedia.org/wiki/Flexible_organic_light-emitting_diode)


100. Which of the following is/are famous for Sun temples?

  1. Arasavalli
  2. Amarakantak‎
  3. Omkareshwar‎

Select the correct answer using the code given below :

  1. 1 only
  2. 2 and 3 only
  3. 1 and 3 only
  4. 1, 2 and 3‎

Sol. Ans (a) A factual question from “Ancient Indian History”. Arasavalli Sun Temple is a 7th-century AD Sun Temple at Arasavalli in Andhra Pradesh, India. No prominent sun temples are present in Amarkantak and Omkareshwar. So statements 2 and 3 are wrong. Hence, answer is (a). (Relevant link : https://en.wikipedia.org/wiki/Suryanarayana_Temple,_Arasavalli)









Questions and Detailed Solutions are being continuously updated ... refresh and check. Comment and let us know your experience, answers and solutions too!